2009 Winter Camp - Buffet Contest

You might also like

You are on page 1of 6

Winter Camp 2009

Buffet contest

A1. Show that for any positive integer n, there exists a positive integer m such that
√ √ √
(1 + 2)n = m + m + 1.

A2. For every ordered pair of positive integers (x, y), define f (x, y) recursively as follows:

 f (x − y, y) + 1 for x > y,
f (x, y) = f (x, y − x) + 1 for y > x,

x for x = y.
For example, f (5, 3) = f (2, 3) + 1 = f (2, 1) + 2 = f (1, 1) + 3 = 4. If f (x, y) ≤ 15, show that
x + y < 2009.
A3. Let x, y, z ≥ 0 be such that x + y + z = 3. Prove that
x3 y3 z3 1 2
3
+ 3
+ 3
≥ + · (xy + yz + zx).
y +8 z +8 x +8 9 27
When does equality hold?
A4. We assign a real number tx,y between 0 and 1 to every point on the plane (x, y) with integer
t +t +t +t
coordinates. This is done in such a way that tx,y = x−1,y x,y−1 4 x+1,y x,y+1 for all x, y. Show
that all the numbers are equal.
C1. A deck contains 52 cards of 4 different suits. Vanya is told the number of cards in each suit.
He picks a card from the deck, guesses its suit, and sets it aside; he repeats until the deck is
exhausted. Show that if Vanya always guesses a suit having no fewer remaining cards than
any other suit, he will guess correctly at least 13 times.
C2. A mathematics competition has n contestants and 5 problems. For each problem, each
contestant is assigned a positive integer score which is at most seven. It turns out every
pair of contestants have at most one problem whose scores are common. Find the maximum
possible value of n.
C3. Let n ≥ 2 be an integer and Tn be the number of non-empty subsets S of {1, 2, . . . , n} with
the property that the average of the elements in S is an integer. Prove that Tn − n is always
even.
C4. For n an odd positive integer, the unit squares of an n × n chessboard are coloured alternately
black and white, with the four corners coloured black. A tromino is an L-shape formed by
three connected unit squares. For which values of n is it possible to cover all the black
squares with non-overlapping trominos? When it is possible, what is the minimum number
of trominos needed?

1
G1. In 4ABC, points D and F are selected on sides BC and AB respectively so that AD · BC =
AB · CF . Let AD and CF intersect at P . Prove that either quadrilateral BF P D is cyclic or
quadrilateral F ACD is cyclic.

G2. Let ABC be a scalene triangle and let A0 , B 0 , and C 0 (respectively) be the points of intersection
of the interior angle bisectors A, B, and C (respectively) with the opposite sides of the triangle.
Now let:

– A00 be the intersection of BC with the perpendicular bisector of AA0 ;


– B 00 be the intersection of CA with the perpendicular bisector of BB 0 ;
– C 00 be the intersection of AB with the perpendicular bisector of CC 0 .

Show that A00 , B 00 , and C 00 are collinear.

G3. Let ω1 and ω2 be concentric circles with ω2 inside ω1 . Let ABCD be a parallelogram with
B, C, D on ω1 and A on ω2 . If BA intersects ω2 again at E and CE intersects ω2 again at
P , prove that CD = P D.

G4. Convex hexagon ABCDEF has area 1. Prove that at least one triangle out of ABC, BCD,
CDE, DEF , EF A, and F AB has area at most 16 .

N1. Find all positive integers n less than 1000 such that n2 is equal to the cube of the sum of its
digits.

N2. Find all integers a, b, c greater than 1 for which ab − 1 is divisible by c, bc − 1 is divisible by
a, and ca − 1 is divisible by b.

N3. The sequence of natural numbers a1 , a2 , a3 , . . . , satisfies the condition an+2 = an+1 an + 1 for
all n. Prove that an − 22 is composite for all n > 10, no matter what a1 and a2 are.

N4. Find all positive integers that can be written in the form

a2 + b2 + 1
ab
where a, b are positive integers.

2
Solutions

√ n
A1. Applying
√ nthe binomial
√ theorem to√(1 n+ 2) , we√ see there exist integers a and b 2such that
(1 2
¡ + √2) = a√+ ¢bn 2 and (1 − 2) = a − b 2. Multiplying these, we get a − 2b =
(1 + 2)(1 − 2) = ±1.
√ √ √ √ √ √
Setting m = min(a2 , 2b2 ), we have m + m + 1 = a2 + 2b2 = a + b 2 = (1 + 2)n .
Source: Romanian Math Stars Competition 2007, #1. Also see CMO 1994, #2.

A2. Let Fn = {1, 1, 2, 3, 5, . . .} be the Fibonacci sequence. We prove by induction on n that


if f (x, y) = n > 1, then min(x, y) ≤ 2Fn−1 and max(x, y) ≤ 2Fn . When n = 2, it is
straightforward to check (x, y) must be one of (2, 2), (1, 2), or (2, 1), and the result holds.
Now assume the result holds for n = k, and consider x, y with f (x, y) = k + 1. If x = y, the
result is trivial. Otherwise, assume without loss of generality that x > y. Then f (x−y, y) = k,
so by our inductive hypothesis, x = max(x − y, y) + min(x − y, y) ≤ 2Fk−1 + 2Fk = 2Fk+1 ,
and y ≤ max(x − y, y) ≤ 2Fk , completing the proof of the inductive step.
It follows that if f (x, y) ≤ 15, then x+y = max(x, y)+min(x, y) ≤ 2F14 +2F15 = 2F16 < 2009.
q
3 y 2 −2y+4
A3. Since y ≥ 0, the AM-GM inequality implies y3x+8 + y+2 27 + 27 ≥ 3 · 3 x3
272
= x3 . Similarly,
y3 z+2 z 2 −2z+4 y z3 x+2 x2 −2x+4
+
z 3 +8 27 + 27 ≥ 3 and x3 +8
+ 27 + 27 ≥ z3 . Adding all three inequalities, we
have:
x3 y3 z3 x + y + z y + z + x 6 y 2 + z 2 + x2
+ + ≥ + − −
y 3 + 8 z 3 + 8 x3 + 8 3 27 9 27
2
4 (x + y + z) − 2xy − 2yz − 2zx
= −
9 27
1 2
= + · (xy + yz + zx).
9 27
2
For equality to hold, we must have y+227 =
y −2y+4
27 =⇒ y 2 − 3y + 2 = 0, so y equals 1 or 2.
The same holds for z and x. Since x + y + z = 3, the only possibility is x = y = z = 1, and
it is easy to check that equality does indeed hold in this case.

A4. Define dx,y,n = tx+n,y+n − tx,y . Let C, ² be constants so that dx,y,1 ≥ C for some x, y
but dx,y,1 ≤ C + ² for all x, y. For any x, y, note that tx,y,n = tx,y,1 + tx+1,y+1,1 + . . . +
tx+n−1,y+n−1,1 ≤ n(C + ²).
We prove by induction that for all n, there exist x, y so that dx,y,n ≥ nC − 3n ². For n = 1, the
claim is trivial. Now suppose the result holds for n, and choose x, y so that dx,y,n ≥ nC − 3n ².
Using the given relation on t, we have:
dx−1,y,n+1 + dx,y−1,n+1 + dx+1,y,n−1 + dx,y+1,n−1
= dx,y,n ≥ nC − 3n ²
4
dx−1,y,n+1 + dx,y−1,n+1
=⇒ ≥ 2nC − 2 · 3n ² − (n − 1)(C + ²) ≥ (n + 1)C − 3n+1 ².
2
Therefore, one of dx−1,y,n+1 or dx,y−1,n+1 is at least (n + 1)C + 3n+1 ², and the claim is proven.
Now suppose dx,y,1 = C > 0 for some x, y. Fix ² > 0 and let m be the largest integer so that
there exists x, y for which dx,y,1 ≥ C + m². Then, as shown above, for each n, there exist

3
x, y so that dx,y,n ≥ nC + m² − 3n ² ≥ nC − 3n ². Choosing n large and then ² small, we have
dx,y,n > 1, which is impossible.
Therefore, dx,y,1 ≤ 0 for all x, y. Similarly, dx,y,1 ≥ 0 for all x, y, and hence tx,y = tx+1,y+1
for all x, y. Similarly, tx,y = tx+1,y−1 for all x, y. The original relation now implies that
tx,y = tx−1,y = tx,y−1 = tx+1,y = tx,y+1 , and the result follows.
Source: Iberoamerican Olympiad, miscellaneous problem
Remark: You cannot assume that there exist x, y for which dx,y,n is maximal. However, there
is a theorem in analysis saying there exist real numbers Mn such that dx,y,n gets arbitrarily
close to Mn without exceeding Mn . If you know this theorem, the proof becomes a lot cleaner.

C1. Let M denote the maximum number of cards remaining in any single suit. As Vanya proceeds,
M will only decrease if the current card is in a suit with M cards remaining, and no other suit
has M cards remaining. In this case, however, Vanya will correctly guess that suit. Therefore,
Vanya will guess correctly every time M decreases.
52
Since M ≥ 4 = 13 initially and it is 0 by the end, Vanya will be correct at least 13 times.
Source: Russia, 1998
§ ¨
C2. There are 7 possible scores on each question. If n ≥ 50, then at least 50
7 = 8 contestants
got the same score on problem 1. But then two of those contestants must have gotten the
same score on problem 2, which is impossible.
Now, for 1 ≤ i, j ≤ 7, let xi,j,k denote the value in {1, 2, . . . , 7} that is congruent to i +
jk (mod 7). Consider 49 contestants Ci,j where contestant Ci,j receives score xi,j,k on problem
k. Suppose that two contestants Ci1 ,j1 and Ci2 ,j2 got the same scores on questions k1 and
k2 . Then i1 − i2 + (j1 − j2 )k1 ≡ i1 − i2 + (j1 − j2 )k2 ≡ 0 (mod 7). Subtracting, we have
(j1 − j2 )(k1 − k2 ) ≡ 0 (mod 7) =⇒ j1 ≡ j2 (mod 7) =⇒ j1 = j2 . But then we must also
have i1 = i2 , which is a contradiction.
Therefore, it is possible to satisfy the required condition with 49 contestants, and hence 49
is the maximum possible value for n.

C3. Let S 0 denote the subsets of {1, 2, . . . , n} with at least two elements and with integer average.
For each set X ∈ S 0 that contains its average x, we pair it with the set X\{x}, and conversely
for each set Y ∈ S 0 that does not contain its average y, we pair it with the set Y ∪ {y}. This
is a proper pairing, so S 0 must contain an even number of sets.
Therefore, Tn has the same parity as the number of singleton sets with integer average, of
which there are exactly n.
Source: Putnam 2002, A3

C4. Let Xn denote the minimum number of trominos required to cover an n × n board in this
2
way. We claim any n ≥ 7 is possible, and Xn = (n+1)
4 .
Indeed, let B denote the set of all squares that are an even number of rows and even number
2
of columns away from the bottom-left square. There are exactly (n+1)
4 such squares, and they
(n+1)2
are all black. Furthermore, each tromino can cover at most one square in B, so Xn ≥ 4 .
(n+1)2
However, if n ∈ {1, 3, 5}, then 3 · 4 > n2 , so it is impossible to place this many trominos
on the board.
2
It remains to show that if n ≥ 7, there exists a valid tiling with (n+1)
4 trominos. For n = 7,
a valid tiling is shown below. Now, suppose there is a valid tiling of an n × n square using

4
2
exactly (n+1)
4 trominos. For k > 1 odd, we can also tile a 2 × k rectangle with k+1
2 trominos
by using two tronominos for the first 2 × 3 rectangle, and 1 tromino for each following 2 × 2
rectangle. Since an (n+2)×(n+2) rectangle can be partitioned into an n×n rectangle, a 2×n
2 (n+3)2
rectangle, and a 2×(n+2) rectangle, it can therefore by tiled with (n+1)
4 + n+1 n+3
2 + 2 = 4
trominos. The result now follows by induction.

Source: IMO Shortlist 2002, C2

G1. Let R denote the circumradius of 4ABC. By the extended sine law, we have AD ·BC = AB ·
4R2 ·(sin A)·(sin B)·(sin C) 2
sin B
sin ADB · BC = sin ∠ADB . Similarly, AB · CF = 4R ·(sinsin
A)·(sin B)·(sin C)
∠CF B . Equating
these, we get sin ∠ADB = sin ∠CF B, which implies ∠ADB = ∠CF B or ∠ADB = 180◦ −
∠CF B. In the former case, F ACD is cyclic; in the latter case, BF P D is cyclic.

G2. Assume without loss of generality that A00 lies on the same side of A0 as C does. Then,
∠A00 AA0 = ∠A00 A0 A = ∠CA0 A = 180◦ − ∠A0 AC − ∠A0 CA = ∠A 2 + ∠B. It follows that
00 00
∠CAA = ∠B, and hence AA is tangent to the circumcircle ω of 4ABC at A. Therefore,
A00 is the intersection of BC and the tangent to ω at A. Similar statements hold for B 00 and
C 00 .
The problem is now equivalent to Pascal’s theorem on the degenerate hexagon AABBCC.
Source: Iberoamerican Olympiad 2004

G3. Let O denote the center of ω1 and ω2 . The perpendicular bisectors of AE and CD are parallel
and both pass through O, so they are in fact identical. Therefore, the quadrilateral AEDC
is symmetric about this line, and BC = AD = EC.
Now, let B 0 and C 0 denote the second intersections of BE and CE with ω1 . ∠EC 0 B 0 = ∠EBC
and ∠B 0 EC 0 = ∠CEB so 4C 0 B 0 E ∼ 4BCE, and hence, C 0 B 0 = EB 0 .
Applying the symmetry argument to B 0 EAB and C 0 EP C, we also have B 0 E = AB = DC
and C 0 E = P C. Also, ∠C 0 EB 0 = ∠P CD since B 0 E and DC are parallel. Therefore,
4C 0 B 0 E ∼
= 4P DC, and the result follows.
Remark: there are two configurations, depending on which of A or E is closer to B, but this
argument works without change in either case.

G4. Let P be the intersection of AD and BE, Q be the intersection of BE and CF , and R be the
intersection of CF and AD. Also assume without loss of generality that P is on the same
side of CF as A and B; i.e., P is between A and R. Then, it is easy to check that R must be
between C and Q, and Q must be between E and P .
In this case, triangles ABR, BCR, CDQ, DEQ, EF P , and F AP are all disjoint, so their
total area is at most 1. It follows that one of them has area at most 16 . Regardless of which
triangle it is, we have found four adjacent vertices P, Q, R, S on the hexagon and a point X on

5
segment P S for which 4QRX has area at most 16 . Note that the area of 4QRX is bounded
between the area of 4QRP and the area of 4QRS. Therefore, one of these triangles also
has area at most 16 , and the result follows.

N1. Let s denote the sum of the digits of n. Then s3 = n2 ≡ s2 (mod 9) =⇒ s2 (s − 1) ≡


0 (mod 9), which implies s ≡ 0 (mod 3) or s ≡ 1 (mod 9). Also, s ≤ 9 + 9 + 9 = 27, and s is
a perfect square since n2 = s3 .
This leaves only the possibilities s = 1 or s = 9, which lead to n = 1 and n = 27, both of
which are valid solutions.
Source: Iberoamerican Olympiad 1999, #1

N2. Note that a, b and c are all relatively prime, since if p|a, b, then a cannot divide bc − 1. Now
the given condition implies:

(ab − 1)(bc − 1)(ca − 1) ≡ 0 (mod abc)


=⇒ a2 b2 c2 − a2 bc − ab2 c − abc2 + ab + bc + ca − 1 ≡ 0 (mod abc)
=⇒ ab + bc + ca ≡ 1 (mod abc)

Since ab + bc + ca > 1, it follows that ab + bc + ca > abc, or equivalently, a1 + 1b + 1c > 1. Now


assume without loss of generality that a ≤ b ≤ c. If a > 2, then since a, b, c are relatively
prime, a1 + 1b + 1c ≤ 13 + 14 + 15 < 1. Therefore, a = 2. If b > 3, then a1 + 1b + 1c ≤ 12 + 15 + 17 < 1.
Therefore, b = 3, which leaves only the option of c = 5.
Conversely, it is easy to check that (a, b, c) = (2, 3, 5) is a valid solution, as are all permutations
of (2, 3, 5).
Source: American Math Olympiad Program 1998

N3. For any n ≥ 3, we have an ≡ 0 (mod an ) and an+1 = an an−1 +1 ≡ 1 (mod an ). We now apply
the recurrence to calculate the sequence {an , an+1 , . . . an+6 } ≡ {0, 1, 1, 2, 3, 7, 22} (mod an ).
Therefore, an+6 − 22 must be a multiple of an .
For n ≥ 3, we have an = an−1 an−2 + 1 > 1. It is also easy to check that an+6 − an > 22.
Therefore, an and an+6
an
−22
are both integers greater than 1, and hence an+6 − 22 is not prime.
Source: American Math Olympiad Program 1998
a2 +b2 +1
N4. Suppose k can be expressed in this form, and let (a, b) be such that ab = k and a + b is
as small as possible.
a2 +1
Suppose a < b. Then b = ka − b is an integer. Denoting this quantity by b0 , we have
(b−1)2 +1
b0 ≤ b < b, and
³ ´2
a2 +1
a2 + (b0 )2 +1 a2 + b +1 a2 + b2 + 1
= = = k,
ab0 a· a2 +1 ab
b

which contradicts the minimality of (a, b). Similarly, b < a is impossible so we must have
2 2 +1
a = b. In this case, a +b
ab is only an integer if a = b = 1 and k = 3.
Therefore, 3 is the only integer that can be expressed in this form.
2 2
Remark: The equation for b0 is found by root-flipping. We interpret a +bab
+1
= k as a quadratic
0
equation in b, and note that if b is one root, then b is the other one.

You might also like